Вы находитесь на странице: 1из 30

Question 1

Solution

i
5

n 1

n 1

i i

i n 5

n 1

i n 2i

2i i i 2 .. i 5 2i i

2i i

1 i
2
1 i

n 1

1 i5
1 i

Correct Option: (3)


Question 2
Solution
Let A, B and C represent z1 , z2 and z3 , respectively, described in a counter clock sense as shown below.

Therefore
z1 z2 BA

cis i
z3 z2 BC 2
( z1 z2 ) 2 ( z3 z2 ) 2
z12 z22 2 z1 z2 z32 z22 2 z2 z3
z12 2 z22 z32 2 z2 ( z1 z3 )

This gives k 2 .
Correct Option: (4)
Question 3
Solution
The coefficients of the given equation are real. One of the roots is 1 + i which implies that both roots must
be imaginary as imaginary roots occur in pairs. This in turn implies that the other root is 1 i. Therefore,
the equations must be
x 2 (1 i 1 i) x (1 i)(1 i ) 0
x2 2 x 2 0

Therefore,
Here,

2015 Wiley India Pvt. Ltd.

b 2, c 2

a = 1 > 0, D = 4 8 = 4 < 0 f ( x) 0 x
Correct Option: (3)
Question 4
Solution
x2 + ax + b = 0 has roots: and , which implies that

x2 ax b ( x )( x )
Similarly,

x2 cx b ( x )( x )
Using the properties of sum and product of the roots, we get

( )( )( )( ) ( 2 c d )( 2 c d )
We also know that,

2 a b 0 2 a b
2 a b 0 2 a b
Replacing the above values in equation (i), we get

( 2 c d )( 2 c d )
[(c a) d b] [(c a) d b]
(c a)2 (c a)(d b)( ) (d b)2

(c a)2 b a(c a)(d b) (d b)2


Correct Option: (4)
Question 5
Solution
The given system of equations can be written as
x 2 y 3 2 z 1 2
2 x 3 y 2 z 2
3x y 2 2 z 1

That is,
1 2
A 2 3
3

3 2

2
2 2

1 2 3
2 3 1
3 1 2
3

6 1 2(4 3) 3(2 9)
5 2 21 18

which implies that |A| 0 and therefore the equations have unique solution.
Correct Option: (1)

2015 Wiley India Pvt. Ltd.

(i)

Question 6
Solution
We have
A1 = (adj A); A2= (adj A1) = adj (adj A); and so on.
Thus,
An adj(adj( (adj A)) ) A

( n 1)n

n times

Note that |A|= |A|.


( n 1)n

Since |An| = k, we have | A |

=k; which implies that


1

|A|= k ( n 1)

Correct Option: (3)


Question 7
Solution
We have
aij (1)i nCi nC j

Consider
a11 = nC1 nC1
a12 = nC1 nC2
a1n = nC1 nCn

(because n is odd)

Again, consider
a11 = nC1 nC1
a21 = nC2 nC1
a31 = nC3 nC1
ann = nCn nCn
nC1 nC1
n n
C2 C1
A nC3 nC1

nC nC
n
1

(because n is odd)
nC1 nCn

n
C2 nCn
nC3 nCn

n
n
Cn Cn

nC1 nC2
n
C2 nC2
nC3 nC2
nC3 nC2

since n is odd. Now,


Trace nC1 nC2 nC3
2

Since

C C C C
2

2015 Wiley India Pvt. Ltd.

nCn 1
2

nCn 0 and n is odd, this implies that


nC1

Correct Option: (3)

nCn 1
2

Question 8
Solution
25! contains 2, 3, 5, 7, 11, 13, 17, 19, 23 as prime factors. By using the result, the highest power of prime
p dividing n! is given by
n n n
p 2 3
p p
25! 222 310 56 73 112 13 17 19 23
Number of factors = (22 + 1) (10 + 1) (6 + 1) (3 + 1) (2 + 1)2 4 = 3,40,032.
Correct Option: (1)

Question 9
Solution
A number is divisible by 3 if and only if the sum of its digits is divisible by 3. Therefore the numerals to
be used are 0, 1, 2, 4, 5 or 1, 2, 3, 4, 5.
In the first case, the number is 5! 4! 96 .
In the second case this is 5! 120 .
Therefore, the required number is 96 120 216 .
Correct Option: (1)
Question 10
Solution
n

r 1 2 3 4
r 1

n(n 1)
2

Now

n(n 1)
2

n 2 ( n 1)!

(n 1) n
1

2 n 2 (n 1) n (n 1)!

1
1

2 n 2 ( n 1)!

1 1 1 1

2 1! 2! 3!
1
(e 1)
2
Correct Option: (1)
Question 11
Solution

2015 Wiley India Pvt. Ltd.

1 1 1

e 1
1 2! 3!

log 2r 4

2
r

Therefore, the summation can be written as


1 2 3
n

2 2 2
2
1
1 n(n 1) n(n 1)
(1 2 3 n)

2
2 2
4

Correct Option: (4)


Question 12
Solution
From summation of geometric series we have.
y xr n 1

Taking log on both sides, we get


log y = log x + (n 1)logr
log y log x
n=
1
log r
Correct Option: (1)
Question 13
Solution
Consider (1 w)20 20 C0 20 C1w 20 C2 w2
Where is w

20 C20 w20

1 i 3
, complex cube root of unity.
2

Therefore,
2

1 3i 20 1 3i 20 1 3i
(1 w) C0 C1
C2
C3

2
2
2

1
Required series is the real part of (1 + w) 20 i.e., w40 or w i.e. .
2
Correct Option: (3)
20

20

20

Question 14
Solution
We have

0 i j n
n

Ci 1 2k

k 0

i 0 j i 1

Ci =

Ci (n i)

i0

= n nC0 + (n 1)nC1 + + 1 nCn1 + 0 nCn


(x + 1)n = nC0 xn + nC1 xn1+ + nCn x0
Differentiating at x = 1, we get
n2n1 > 1 + 2n+1 1

2015 Wiley India Pvt. Ltd.

20

1 i 3
C20

20

where n > 4. Therefore, the least value is 5.


Correct Option: (2)
Question 15
Solution
x 2 nx

n
0
2

For roots to be real D 0


4n
0 n(n 2) 0
2
Therefore,
n 0, n 2
Therefore, favorable values of x = {2, 3, 4, 5, 6, 7, 8}
7
Therefore, the required probability =
8
Correct Option: (1)
n2

Question 16
Solution
1
where B1 is the event of choosing bag 1
3
1
P(B2) = where B2 is the event of choosing bag 2
3
1
P(B3) = where B3 is the event of choosing bag 3
3
B = Event in which black ball is drawn
1 7

7
B3 P B B3
3 10
P

1 7 1 1 1 3 15
P( B)
B

3 10 3 5 3 5
Where P(B) = P(B B1) + P(B B2) + P(B B3)
Correct Option: (1)

P(B1) =

Question 17
Solution
5

1 1 1
Required probability= 10C5
2 2 2
10
C
= 115
2

Correct Option: (2)


Question 18
Solution
Reciprocal system is given by

2015 Wiley India Pvt. Ltd.

b c
c a
a b
, b
, c
[abc ]
[abc ]
[abc ]

That is,

b c c a a b
[a b c]

[a b c ] [a b c ] [a b c ]
1

[a b b c c a ]
[a b c ]3
1

as [a b b c c a ] [abc ]2
[a b c ]
Applying AM GM, we get
9
4 a b c
a b c

4 a b c 9

a b c
2

4 a b c + 9[a' b' c' ] 12

Correct Option: (2)


Question 19
Solution

a b b c c a (a b c)
Taking dot product with a both sides

1
1

[a b c] ( 2)2 ( 2)2 ( 2)2


2
2

= (2 + 1 + 1)
1
[a b c]
4
aa ba ca

2 1 1
[ a b c] a b b b c b 1 2 1 4
ac bc cc 1 1 2
2

[a b c] 2
1

2
Correct Option: (4)
Question 20
Solution

[a 2b 2b 3c 3c a] 2 6 3[a b b c c c ]
36[a b b c c a]
36[a b c]2

2015 Wiley India Pvt. Ltd.

Correct Option: (3)


Question 21
Solution
p
T
T
F
F

q
T
F
T
F

pq
T
T
T
F

~p
F
F
T
T

(p q) ~ p
F
F
T
F

~pq
F
F
T
F

Correct Option: (4)


Question 22
Solution
Let us make the truth table,
p
T
T
F
F

q
T
F
T
F

pq
T
T
T
F

(p q) q
T
T
F
T

(q p)
T
T
F
T

p (q p)
T
T
F
T

(p q)
T
F
T
T

p (p q)
T
T
T
T

p (p q)
T
F
T
T

Hence, p (p q) is a tautology.
Correct Option: (3)
Question 23
Solution
The mean of given probability distribution is given by
xi pi 0 0.17 + 1 0.29 + 2 0.27 + 3 0.16 + 4 0.07 + 5 0.03 + 6 0.01
= 0.29 + 0.54 + 0.48 + 0.28 + 0.15 + 0.06 =1.80
Note: 0.17 + 0.29 + 0.27 + 0.16 + 0.07 + 0.03 + 0.01 = 1, that is, f ( xi ) 1.
Correct Option: (2)

Question 24
Solution
xi
1
Variance xi2

n
n

50

(2r 1)
50
1

(2r 1) 2 r 1
50 r 1
50

2015 Wiley India Pvt. Ltd.

1
12 22
50

1 3 5 992
992

50

1 2 2
1 3 52

50

100 2 4
2

502
100 )

50

1 100 101 201 22 50 51 101

2500
50
6
6

= 101 67 2 17 101 2500


= 101 33 2500 = 833

Correct Option: (4)


Question 25
Solution
We have
abc
(1 cos A)
s
abc b2 c 2 a 2

1 2bc
s

abc (b c)2 a 2
2bc
s

abc (a b c)(b c a)

s
2bc

1
[2(abc)]
a(b c a)
2abc
a(b c a) a(b c a) b(c a b) c(a b c)

2bc 2ca 2ab a 2 b 2 c 2


2bc 2ca 2ab (a 2 b 2 c 2 )

Therefore, k 1 .
Correct Option: (1)
Question 26
Solution
With the given perimeter, the maximum area of the triangle is possible if it is an equilateral triangle. So
A = B = C = 60
Now circumradius
a
R
(a 3)
2sin A
R

So circumradius is 3 .
Correct Option: (2)
Question 27

2015 Wiley India Pvt. Ltd.

3
3 2

3
2 sin 60 2
3

Solution
Whenever 1 sin x < 0, we have

2cos x 0 x n

That is, if x

1
y [sin x 1]
2
1
([sin x] 1)
2
1
(1 1) 1
2
, n I

,cos( x) 0 and sin x = 1, which implies that both equations are satisfied.
2

This implies that if x 2n

, cos x 0 and sin x = 1; n I Both equations are satisfied.


2
Hence, there are infinite solutions.
Correct Option: (4)

Question 28
Solution
We have
tan x 1

tan y 4
sin x cos y 1

cos x sin y 4
cos x sin y

Therefore, from sin x cos y

4
5

1
sin x cos y
5

1
4
and cos x sin y , we have
5
5

sin( x y) 1 and sin( x y )

Therefore,
x y n (1)n

and

3
5

x y k (1)k sin 1

(3)
5

Correct Option: (1)


Question 29
Solution
Number of solution is given by the number of points of intersection of these two graphs:
y = 4cosx and y = 5|x|
Draw these two graphs on the same axes.

2015 Wiley India Pvt. Ltd.

There are infinite many intersections, so the number of solutions will be infinite.
Correct Option: (4)
Question 30
Solution
It is known that the sum of the interior angles of an n-sided convex polygon is (2n 4) right angles.
Therefore,
cos( Ak 1 Ak 2 An ) cos[(n 2) ( A1 A2 Ak )] cos( A1 A2 Ak )
Now,
cos( A1 A2 Ak )
1
cos( Ak 1 Ak 2 An )
depending on if n is even or odd. Hence, the value of

cos( A1 A2 Ak )
is independent of k but
cos( Ak 1 Ak 2 An )

depends on n.
Correct Option: (2)
Question 31
Solution
Writing the domains of the individual functions,
Function
sin 3x
cos 4x
tan x
cot x
sec x
cosec x

Domain
R
R
R (2n + 1)/2
R n
R (2n + 1)/2
R n

Now D1 D2 D3 D4 D5 D6 = R n/2, n I
Correct Option: (4)
Question 32
Solution
According to the question,

2015 Wiley India Pvt. Ltd.

Perimeter of the sector, P1 = 2r + l

Perimeter of the circle, P2 = 2r


P1 1
2r l 1

P2 2
2r
2
2r + l = r
l = r 2r
l = r( 2)
l
2
r
l

r
Angle measure = ( 2) radians

Correct Option: (1)


Question 33
Solution
We have x 2 9 only if x 3 . For this value of x, the equation 2 x 4 is not satisfied. Sets in options
(1), (3), and (4) are non-empty.
Correct Option: (2)
Question 34
Solution
The relation is not reflective and transitive but it is symmetric because.
x2 + y2 = 1 y2 + x2 = 1
Correct Option: (2)
Question 35
Solution

2015 Wiley India Pvt. Ltd.

Since
8n 7n 1 = (7 + 1)n 7n 1
n
n
n1
n
= 7 + C1 7
+ C2 7n 2 + ....nCn 1 7 + nCn 7n 1
= nC2 72 + nC3 73 + . + nCn 7n ( nC0 = nCn, nC1 = nCn 1 etc.)
= 49[nC2 + nC3 (7) + .... + nCn 7n 2]
Therefore
8n 7n 1 is a multiple of 49 for n 2.
n
For n = 1, 8 7n 1 = 8 7 1 = 0
For n = 2, 8n 7n 1 = 64 14 1 = 49
Therefore, 8n 7n 1 is a multiple of 49 for all n N.
Therefore, X Y.
Correct Option: (1)
Question 36
Solution
We have,

lim

n(n 1)(2n 1)

6n
13 23 33
lim
n
n4

2 1

6 3
n3
12 22 32
lim
n4
n

n 2 (n 1) 2
n(n 1)(2n 1)
lim
n
n

4
6n 4
1
0
4
3 4 .
lim

Therefore,
Correct Option: (4)
Question 37
Solution
Consider

x3
e x

as x ,

x3
will tend to 0 from negative side as x < 0.
e x
x3
x will tend to 1
e

x3
lim x 1
x e

Correct Option: (2)
Question 38
Solution
Given relation can be re-written as

2015 Wiley India Pvt. Ltd.

n2

x
f ( x) f f
2

Replacing x with

x x x
, , ,
2 22 23

x
2n

x
x 2
f x
2
4

on both sides and adding all the subsequent equations, we have

1 n 1
1
x
x
x 2 22
.(1)
f ( x) f n 1 f f n 2 x

1
2
2
2
1

22
Since f is continuous, taking limits on both sides of Equation (1) as n ,we get

4
x
f ( x) f (0) f f (0) x 2
2
3

Therefore,
x 4
f ( x) f x 2
2 3
x x
In Equation (2), again replace x with , 2 ,
2 2

.(2)
,

x
2n

and add all of them. Then, we have

1 n 1
1 2
x 4
f ( x) f n 1 x 2 2

2 3 1 1

Again, taking limits both sides as n , we get


f ( x) f (0)

16 2
x
9

f ( x) f (0)

16 2
x
9

Therefore,

Correct Option: (4)


Question 39
Solution

f ( x) [ x] {x} n x n n x n 1
Graphing this function, we get,

2015 Wiley India Pvt. Ltd.

Clearly is continuous in R but not differentiable R as at every integer point right hand derivative does not
exist
Correct Option: (2)
Question 40
Solution
Let y x = 2t. Substituting in y = t2 + t + 1, we get
2

yx
yx
y
1

2
2

4 y y 2 x2 2 xy 2 y 2 x 4
y 2 x2 2 xy 2 y 2 x 4 0
Differentiating with respect to x, we get
dy
dy
dy
2 y 2x 2x 2 y 2 2 0
dx
dx
dx
dy
( y x 1) 1 y x
dx
Therefore,
dy 1 y x

dx y x 1
Correct Option: (3)
Question 41
Solution
We have
1 1
y

x
x

Therefore,

2015 Wiley India Pvt. Ltd.

(1/2)

dy d 1 1

dx dx x
x

(1/2)

1 1 1

x
2 x

(1/2) 1

1 1 1

x
2 x

(3/2)

d 1
(1/2)
( x)

dx x

1 1 (3/2)
2 ( x)

2
x

If x = 4, we get
dy 1 1 1

dx 2 4 2
3

1 3 2

2 4

(3/2)

1 1 1
(1)
16 2 8

1

8

That is,
dy 1 4 2 1

dx 2 3 3 8
1

6 3

Correct Option: (4)


Question 42
Solution
It is given that y 2 P( x) . In this problem, for simplicity, we denote dy / dx by y1 and d 2 y / dx 2 by y2 .
Differentiating the given equation y 2 P( x) with respect to x, we have
.. (1)
2 yy1 P( x)
Again differentiating both sides with respect to x , we get
2 y12 2 yy2 P( x)
..(2)
2
Multiplying both sides of Equation (2) with y , we get
2 y 2 y12 2 y3 y2 y 2 P( x) P( x) P( x)
Therefore,
2 y3 y2 P( x) P( x) 2 y 2 y12
1
P( x) P ( x) ( P ( x))2
[By Eq. (1)]
2
So
d
1
2 ( y3 y2 ) P( x) P( x) P( x) P( x) 2P( x) P( x)
dx
2
P( x) P( x)
Correct Option: (3)

2015 Wiley India Pvt. Ltd.

Question 43
Solution
Consider the slope of a point P(x, y) on the curve and origin.

y0
x0
[ x3 / (1 x 4 )] 0

x0
2
x

1 x4
g ( x)
(say)

m Slope

Maximizing g(x), we get

g ( x)

(1 x 4 )(2 x) x 2 (4 x3 )
(1 x 4 )2
(2 x)(1 x 4 )
(1 x 4 )2

For local maxima and minima,


g(x) = 0
Therefore,

x4 1
x2 1
x 1
Correct Option: (3)
Question 44
Solution
We know that
x2 0
x2 2 2
(x2 2)5 (2)5
(x2 2)5 32
(x2 2)5 + 32 0
5
5
3( x 2) 32 30
5
5
3( x 2) 32 1
So, the minimum value of the function is 1.
Correct Option: (2)
Question 45
Solution

2015 Wiley India Pvt. Ltd.

x = a sec

y = a tan
x y = a2(sec2 tan2)
x2 y2 = a2
c
x , y c

xy = c2
dy
yx 0
dx
2

Now
dy
0
dx
dy y

dx
x
dy x

dx y
x
m1
y
y
m2
x
m1m2 = 1

2x 2 y

So angle between curves is 90.


Correct Option: (3)
Question 46
Solution
Let l be the length of the cone, h be the height of the cone and r be the radius of the upper part of cone.
The cone formed by the circular sheet of radius R is related as follows:
lR
h2 r 2 R 2

Volume of the cone is


1
V r 2h
3

Therefore,
V2

2
9

2
9

r 4 h2
r 4 (R2 r 2 )

Thus,
f (r ) r 4 ( R 2 r 2 )
f '(r ) 4 R r 6r
2 3

f "(r ) 12 R 2 r 2 30r 4

For the local maxima and local minima, we get


That is,
2015 Wiley India Pvt. Ltd.

f '(r ) 0

(say)

4 R 2 r 3 6r 5 0
2r 3 (2 R 2 3r 2 ) 0
r0

2
r
R
3

or

Therefore, f " R is negative and the volume is maximum. Now,


3
2
2
R 2 R 2 R 2
3
3 3

1/2

2
at r R
3

2 R 2 1
R

9
3
2 R3
9 3

from which we conclude that k = 9.


Note: Absolute maximum does not occur at end points.
Correct Option: (4)
Question 47
Solution
I

dx
( x 1)( x 1)( x 1)
( x 1) dx
2

x 1
x 1

( x 2 1)( x 2 1)( x 4 1)
( x 1) dx
4
I1 I 2
( x 1)( x 4 1)

Therefore,
I1

x
( x 1)( x 4 1)
4

dx

Substituting x2 t 2 x dx dt . Therefore,
I1

1
dt

2
2 (t 1)(t 2 1)

1 1
1 1
2
2
dt

2 t 1 t 1 2
1 dt
1 dt
2
2
4 t 1 4 t 1

1 1 t 1 1
1
ln
tan t c1
4 2 t 1 4
1 x2 1 1
ln 2
tan 1 ( x 2 ) c1
8 x 1 4

and

2015 Wiley India Pvt. Ltd.

I2

1
( x 1)( x 4 1)
4

dx

1 1
1
4
4
dx
2 x 1 x 1

Therefore,
I2

1
dx
1
dx

2 ( x 2 1)( x 2 1) 2 x 4 1

1 1
1 dx 1
K

g ( x)
2 x2 1 x2 1 2 2
2

1 1 1 x 1 tan 1 x 1
K

ln
g ( x)
2 2 2 x 1
2 2
2

Now,
I I1 I 2
1 x2 1 1
1 x 1 1
1
ln 2
tan 1 ( x 2 ) ln
tan 1 x g ( x) c
8 x 1 4
8 x 1 4
2

Correct Option: (2)


Question 48
Solution
x 2014 1
ex
dx
2
( x 2014)

x
1
1

e dx
2
x 2014 ( x 2014)
From e x f ( x) f ( x) dx e x for

ex
C
x 2014

Correct Option: (4)


Question 49
Solution
I x2 e x dx x ( xe x ) dx
2

Let us consider
J x e x dx
2

Substituting x 2 t , we get
2 x dx dt

That is,
1 t
1 e t e t
e x
e dt

2
2 1
2
2

Now,

2015 Wiley India Pvt. Ltd.

2
I x ( xe x ) dx
I
II

x2
e
x
2

x2
1 e

dx

xe x 1 x2

e dx
2
2
2

(integrating by parts)

Using limits, we get


xe x2
I
2

2
1
e x dx
0 2 0

2 2
4

Note 1: We can write


lim xe x lim
2

x e x

form

lim

1
2 xe x

(using L Hospital rule)

Note 2: We have written integrand x 2 e x as x x e x so that x e x can be integrated and we can use
integration by parts and use limits in the last step.
Correct Option: (4)
2

Question 50
Solution
Let
e3

dx
x 1 log e x

I
1

Put loge x t . Therefore x


Now

et dt .

e so that dx

t
3

0
t

So
3

e 1

2015 Wiley India Pvt. Ltd.

dt
1 t

et dt

(1

t ) (1/ 2) 1
(1/ 2) 1
1

2[ 1

2(2
2

3
0

1]

1)

Correct Option: (1)


Question 51
Solution
We have f (0)

f (2)

0 and
2

3
4

f ( x) dx
0

Now,
2

x f ( x) dx

x f ( x)

2
0

f ( x) dx

2 f (2)

3
4

3
4

0
3
4

Correct Option: (3)


Question 52
Solution
Graphing the two functions,

4 (cos x sin x)dx


0

2015 Wiley India Pvt. Ltd.

/4

4 sin x cos x 0
4

2 1

Correct Option: (3)


Question 53
Solution

K 2 f ( x) 4K f '( x) 4 f "( x) 0
It is quadratic in K. Its roots are equal. Therefore,

D 0 (4 f '( x))2 4[ f ( x) 4 f ''( x)]


16 f ' x 16 f x f " x 0
2

f ' x f x . f " x
2

f ' x

f " x

f x
f ' x
Integrating on both sides, we get

ln f x ln f ' x ln c

ln f x ln cf ' x

f x cf ' x
f x cf ' x
f x af ' x

.(1)
where a is a constant (suppose a = c, then a is a constant). Substituting x = 0, we get
f 0 af 0

1
3
Therefore, Equation. (1) becomes
1 3a a

f x
3 f x f ' x

f ' x
f x

1
f ' x
3

Integrating with respect to x, we get


f x e3x c2 ec2 e3x c3e3x
f x c3 e3 x

f x c4 e3x

where c

ln f x 3x c2
(ec2 is a constant)

c3 e

c2

etc

Substituting x = 0, we get
f 0 1 c4e0 c4 c4 1

f x e3x

2015 Wiley India Pvt. Ltd.

Correct Option: (3)


Question 54
Solution
Given that
dy
1

dx [(3 / 2) x] y 2
dx 3

x y2
dy 2

dx 3
x y2
dy 2

(1)

(3/2)dy
I.F. e
e(3/2) y

Multiplying Equation (1) by e( 3/2) y , we get


dx 3
e(3/2) y e(3 y /2) x e(3 y /2) y 2
dy 2
d

xe(3 y /2) y 2e(3 y /2)


dy

xe(3 y /2) y 2e(3 y /2) dy

Let us consider that


J y 2e(3 y /2) dy
3y
Substituting t , we get
2
2
dy dt
3
Therefore,
J

4 2 t 2 8 2 t
t e dt
t e dt
9
3 27

Let us also consider that


K t 2 et dt
I II

t 2et 2 t et dt
I II
2
t
t
t e 2(t e et dt )
t 2 et 2t et 2 et c
2
3 y (3 y /2)
3 y (3 y /2)

2
2e(3 y /2) c '
e
e
2
2

Therefore,
3 y
3 y
3 y

8 9 2 2
2
J
3 ye
2e 2 c
y e

27 4

2015 Wiley India Pvt. Ltd.

3 y
3 y
3
2 2 2 y 8
16
y e
ye 2 e 2 c
3
9
27

Thus,

3 y
xe 2 J

3y
2
8
16
x y2 y
ce 2
3
9
27

Correct Option: (4)


Question 55
Solution
Let P be the middle point of the line segment joining A(3, 2) and B(1, 1).
Q

A (3, 1)

B (1, 1)

Then
3 1
1 1
= (2, 0)
,
2
2
Let P be shifted to Q where PQ = 2 and y co-ordinate of Q is greater than P.
Now slope of AB = 1
Therefore, slope of PQ = 1
Co-ordinates of Q by distance formula = (2 2 cos , 0 2 sin ) where tan = 1
1
1

, 0 2
= 2 2
= (2 2 , 2 )
2
2

as y co-ordinates of Q is greater than that of P.


Hence Q = (2 + 2 , 2 ) is the required point.
Correct Option: (2)

P=

Question 56
Solution
In the given equation, a = 2, h = 2, b = 5, g = 10, f = 11, c = 14. Therefore,
ab h2 = 10 4 = 14
hf bg = 2(11) (5)(10) = 22 + 50 = 28
gh af = 10(2) 2(11) = 42
Therefore,
hf bg gh af
New origin
,

ab h 2 ab h 2
28 42

14 14
(2, 3)

Correct Option: (4)

2015 Wiley India Pvt. Ltd.

Question 57
Solution

e = 2.71 [e] = 2
[{e}] = 0
logee = 1
So equation of straight line passing through (1, 0) having slope 2 is
y y1 = m(x x1)
y 0 = 2(x 1)
2x y = 2
Correct Option: (3)
Question 58
Solution

After reflection, it will make equal angles with x-axis, So by symmetry reflection of P w.r.t. x-axis lies on
the line RQ.
Equation of straight line passing through the points (1, 3) and (0,3) is,

y 3

Now putting y = 0 x = 1/2


So point on x-axis is (1/2, 0).
Correct Option: (1)
Question 59
Solution

2015 Wiley India Pvt. Ltd.

3 (3)
( x 1)
0 (1)

y + 3 = 6(x + 1)
6x y +3 =0

Correct Option: (3)


Question 60
Solution
Let y = m(x 13) be a line through (13, 0).
This line touches the circle, which implies that,
m(0 13) 0

5
1 m2
169m 2 25(1 m 2 )
144m 2 25
5
m
12

Therefore, the angle between the tangents is


5
2 tan -1
12

Correct Option: (1)


Question 61
Solution
Let P be (x1, y1) so that
x1 4a 0

(1)
2

The chord of contacts of P(x1, y1) with respect to y = 4ax is


y y1 2a( x x1 ) 0

y y1 2ax 8a 2 0. [ x1 4a,fromEq.(1)]

2ax y1 y

1
8a 2
Hence, the combined equation of the pair of lines OQ and OR is

2ax y1 y
y 2 4ax
0
8a 2

In this equation, we have

2015 Wiley India Pvt. Ltd.

(i.e., homogenize the curve with the help of line)

Coefficient of x2 + Coefficient of y2 = 0
Therefore, QOR 90 .
Quick Tip: The chord of contact of point P(x1, y1) with respect to parabola y2 4ax = 0 is
y1y 2a(x + x1) = 0

If the combines equation of OQ and OR is obtained by homogenizing the equation of the parabola with

the help of the line QR then QOR if the coefficient of x2 + coefficient of y2 = 0 (in this equation).
2
Correct Option: (4)
Question 62
Solution
Let the ellipse be
x2
a2

y2
b2

This passes through (3, 1) and (2, 2).


Therefore,
9 1
1
a 2 b2
4 4
1
a 2 b2
Solving Equation (1) and (2) for a2 and b2, we get

(1)
(2)

a2

2015 Wiley India Pvt. Ltd.

32
3

b2

and

32
5

Therefore

b2 a 2 (1 e2 ) e

2
5

Correct Option: (3)


Question 63
Solution
The denominator of the given function is x10 x9 + x8 x7 + 1.

x 1; x10 x9 ; x8 x7 . Therefore, the denominator of the given function is positive.


x 0 , each term of the denominator is positive.
0 x 1 , the denominator is

x9 (x 1) + x7(x 1) + 1 = x10 + x8 (1 x) + 1 x7
which is positive and therefore each term is positive.
At x = 0, the denominator is 1. Therefore, there is no value of x for which f (x) is not defined.
Therefore, Df = R.
Correct Option: (1)
Question 64
Solution
Here f ( x) is defined for all real x, since e x e|x| 0 for all x . Also

for x 0
0
x x
f ( x) e e
.
e2 x 1
e x e x e2 x 1 for x 0

Therefore

f ( x) 1
For x 0,

2
for all x 0
e 1
2x

2
1
e 1
From this it follows that the range of f is (1,0] .
y f ( x) 0

Correct Option: (2)


Question 65
Solution

2015 Wiley India Pvt. Ltd.

y 1

2x

(i) sin 1 sin

sin 1 sin
3
3

sin 1 sin

3
2 2

(ii) cos cos


3
3
2

tan 1 tan
(iii) tan 1 tan
3
3

tan 1 tan
3 3

2
2
2
cos 1 cos
tan 1 tan
3
3
3
2 2


3 3 3
3

sin 1 sin

Now

Correct Option: (3)


Question 66
Solution
Put Sin 1 (1/ 5) and Cos1 x . Therefore
1
sin and cos x
5
Now
1 sin( )
1 sin cos cos sin
1
24
1 ( x)
1 x2
5
5
5 x 24 1 x 2
(5 x) 2 24(1 x 2 )
25 x 2 10 x 1 0
(5 x 1)2 0
x

Correct Option: (4)

2015 Wiley India Pvt. Ltd.

1
5

Вам также может понравиться